Convergence of Integral with Variable Limits: Investigating a and b

In summary, LCKurtz has been very efficient and helpful in this thread. He always is efficient and helpful. I don't really appreciate it when you attack a member who is trying to help you. Please don't do this again.
  • #1
peripatein
880
0
Hi,

Homework Statement


For what values of a and b is the integral in the attachment convergent?


Homework Equations





The Attempt at a Solution


By the comparison test, as well as the fact that arctanx/x diverges, I believe a-b<1. Is that correct?
 

Attachments

  • Integral.jpg
    Integral.jpg
    2.8 KB · Views: 323
Physics news on Phys.org
  • #2
peripatein said:
Hi,

Homework Statement


For what values of a and b is the integral in the attachment convergent?


Homework Equations





The Attempt at a Solution


By the comparison test, as well as the fact that arctanx/x diverges, I believe a-b<1. Is that correct?
Here's the integral, in LaTeX:
$$ \int_1^{\infty} \frac{x^a~arctan(x)~dx}{2 + x^b}$$
 
  • #3
peripatein said:
Hi,

Homework Statement


For what values of a and b is the integral in the attachment convergent?


Homework Equations





The Attempt at a Solution


By the comparison test, as well as the fact that arctanx/x diverges, I believe a-b<1. Is that correct?

No, that is not correct. For example a = 0 and b = 1. You might try making use of the fact that arctan(x) is bounded.
 
  • #4
I meant to write, a-b<-1.
 
  • #5
peripatein said:
I meant to write, a-b<-1.

I get that too, assuming a and b are nonnegative. Is that given? You didn't show your argument though.
 
  • #6
It is not given that a and b are nonnegative.
 
  • #7
So you have more to do...
 
  • #8
My argument was that it had to be greater than or equal to (pi/4)/(x^p), where p is greater than 1, and less than or equal to (pi/2)/(x^p), as then, by the comparison test, the original integral would converge.
 
  • #9
Sorry, but to continue this discussion, I need to see your work, showing assumptions on a and b, your equations/inequalities, and your conclusions or where you are stuck. Unfortunately, I have to leave for a few hours, but that will give you time to post the details.
 
  • #10
Wouldn't the inequality be as I stated above just before you were leaving, namely:
(π/4)/xb-a<arctanx/xb-a<(π/2)/xb-a
Hence, for the integral to converge then by the comparison test b-a>1?
Why is that incomplete/incorrect?
 
  • #11
peripatein said:
Wouldn't the inequality be as I stated above just before you were leaving, namely:
(π/4)/xb-a<arctanx/xb-a<(π/2)/xb-a
Hence, for the integral to converge then by the comparison test b-a>1?
Why is that incomplete/incorrect?

Let's be clear about something. You did not state that inequality anywhere before the post to which I am responding. And you still haven't shown any work which can be analyzed to see what you are thinking. And the reason that is incomplete/incorrect is because it is incomplete.

For example, if a=-2 and b = -3 you don't have b-a>1 yet the integral converges. So until you post more complete arguments as I have suggested in post #9 instead of just answers, we don't have anything to discuss.
 
  • #12
First of all, I DID post that just before you were leaving. You should check carefully and read posts well before you accuse anyone of anything or make any demands/claims.
Second, there is no need to be rude and use such tone. If you don't wish to help in a civil manner, do me a big favour and don't!
 
  • #13
peripatein said:
First of all, I DID post that just before you were leaving. You should check carefully and read posts well before you accuse anyone of anything or make any demands/claims.

I don't see it either...

Second, there is no need to be rude and use such tone. If you don't wish to help in a civil manner, do me a big favour and don't!

He was not being rude. He is merely stating the rules of this forum. That is: you must show your work and your thinking process. This is meant to make our help more effective. If you show all your work, then it's much more easy for us to see where you're going wrong and how to help you.
 
  • #14
What about this post then, posted at 21:24: "My argument was that it had to be greater than or equal to (pi/4)/(x^p), where p is greater than 1, and less than or equal to (pi/2)/(x^p), as then, by the comparison test, the original integral would converge."
??
It was not typed using LaTex, but it was there all along and was precisely what I wrote again for him to read upon his return.
 
  • #15
And if you do not consider that good enough an answer, it is not because I am holding back something, but because that is all I know and all I am capable of deriving myself at this stage. He could have tried being more efficient and helpful, instead of making ridiculous assertions and demands.
 
  • #16
peripatein said:
And if you do not consider that good enough an answer, it is not because I am holding back something, but because that is all I know and all I am capable of deriving myself at this stage. He could have tried being more efficient and helpful, instead of making ridiculous assertions and demands.

LCKurtz has been very efficient and helpful in this thread. He always is efficient and helpful. I don't really appreciate it when you attack a member who is trying to help you. Please don't do this again.

What we want of you is a clean argument.
For example

My argument was that it had to be greater than or equal to (pi/4)/(x^p), where p is greater than 1, and less than or equal to (pi/2)/(x^p),

Write out the actual inequality and prove it.

as then, by the comparison test, the original integral would converge.

Show this explicitely.
 
  • #17
First of all, as I am experiencing difficulties with my keyboard you would have to excuse my not using LaTex at this point. It is certainly not my choice.
Regarding my argument:
I realized that arctanx/(x^p) does not converge unless p were greater than 1. Hence, I was trying to express the original expression as arctanx/(x^p) and demand that p is greater than 1. In order to make sure the original expression converges, I needed to find something convergent which is greater (following the rationale of the comparison test). So I noted that arctanx was bounded from above by pi/2, and so I wrote that the original expression was < (pi/2)(x^a)/[2+x^b] < (pi/2)/(x^(b-a)). I then demanded b-a to be greater than 1.
I realize that this answer might be/is incomplete, but that is all I can muster on my own at this point.
Now if you care to help, I'd appreciate it.
 
  • #18
Well why don't you see why your argument doesn't work for the example LCKurtz gave back in post #11?
 
  • #19
I do see that, but am not sure how to correct it, which is why my last post asked for assistance.
 
  • #20
Examine each step of your argument to see where and why it breaks down. LCKurtz has already given you a hint, noting that your basic argument works when a and b are non-negative. Find which step doesn't work if you drop that assumption. That might give you ideas of how to fix up your proof.
 

1. Is it possible to determine if an integral is convergent without calculating it?

Yes, there are various tests and criteria that can be used to determine the convergence of an integral, such as the comparison test, ratio test, and integral test.

2. What is the difference between a convergent and divergent integral?

A convergent integral has a finite value when evaluated, while a divergent integral does not. In other words, a convergent integral leads to a definite answer, while a divergent integral does not have a definite value.

3. Can an integral be both convergent and divergent?

No, an integral can only be either convergent or divergent. It cannot be both at the same time.

4. How does the function being integrated affect the convergence of an integral?

The function being integrated can have a significant impact on the convergence of an integral. Generally, if the function approaches zero as x approaches infinity, the integral is more likely to be convergent. On the other hand, if the function approaches a non-zero value or infinity as x approaches infinity, the integral is more likely to be divergent.

5. Can the bounds of integration affect the convergence of an integral?

Yes, the bounds of integration can greatly affect the convergence of an integral. For example, if the bounds of integration are infinite, the integral may be more likely to be divergent. If the bounds are finite, the integral may be more likely to be convergent.

Similar threads

  • Calculus and Beyond Homework Help
Replies
5
Views
1K
  • Calculus and Beyond Homework Help
Replies
4
Views
1K
  • Calculus and Beyond Homework Help
Replies
4
Views
286
  • Calculus and Beyond Homework Help
Replies
5
Views
1K
  • Calculus and Beyond Homework Help
Replies
16
Views
3K
  • Calculus and Beyond Homework Help
Replies
2
Views
830
  • Calculus and Beyond Homework Help
Replies
14
Views
1K
  • Calculus and Beyond Homework Help
Replies
7
Views
2K
  • Calculus and Beyond Homework Help
Replies
8
Views
651
  • Calculus and Beyond Homework Help
Replies
1
Views
232
Back
Top